Time limit: 0
Quiz Summary
0 of 38 Questions completed
Questions:
Information
You have already completed the quiz before. Hence you can not start it again.
Quiz is loading…
You must sign in or sign up to start the quiz.
You must first complete the following:
Results
Quiz complete. Results are being recorded.
Results
0 of 38 Questions answered correctly
Your time:
Time has elapsed
You have reached 0 of 0 point(s), (0)
Earned Point(s): 0 of 0, (0)
0 Essay(s) Pending (Possible Point(s): 0)
Average score |
|
Your score |
|
Categories
- NBME 7 BLOCK 4 0%
Would you like to submit your quiz result to the leaderboard?
Loading
Pos. | Name | Entered on | Points | Result |
---|---|---|---|---|
Table is loading | ||||
No data available | ||||
- 1
- 2
- 3
- 4
- 5
- 6
- 7
- 8
- 9
- 10
- 11
- 12
- 13
- 14
- 15
- 16
- 17
- 18
- 19
- 20
- 21
- 22
- 23
- 24
- 25
- 26
- 27
- 28
- 29
- 30
- 31
- 32
- 33
- 34
- 35
- 36
- 37
- 38
- Current
- Review
- Answered
- Correct
- Incorrect
-
Question 1 of 38
1. Question
A 23-year-old man is evaluated for excessive bruising since childhood. He says some of his family members “bleed easily” and sometimes require blood transfusions. Temperature is 36.7 C (98.1 F). Physical examination is unremarkable. Complete blood cell count is unremarkable. Platelet aggregometry is abnormal. Genetic testing identifies a defect in a platelet surface glycoprotein receptor that normally binds fibrinogen to support platelet aggregation. The abnormal glycoprotein receptor found in this patient is most likely targeted by which of the following medications?
CorrectIncorrect -
Question 2 of 38
2. Question
A 23-year-old woman comes to the emergency department with vaginal bleeding and lower abdominal cramps. Her last menstrual period was 6 weeks ago, and a home pregnancy test was positive. She has no medical history and does not use tobacco or alcohol. Pelvic ultrasonography reveals a gestational sac in the left fallopian tube without evidence of rupture. A single dose of methotrexate is administered to treat her condition. Which of the following substances will most likely accumulate in embryonic tissues as a result of treatment?
CorrectIncorrect -
Question 3 of 38
3. Question
An 18-year-old man with a history of recurrent hemarthroses and severe mucosal bleeding due to hemophilia A comes to the office for follow-up. He has had occasional breakthrough bleeding despite receiving prophylactic factor VIII infusions. Testing reveals the patient now has inhibitor alloantibodies against factor VIII, and factor replacement products can no longer be used. Treatment with a novel medication that prevents bleeding by mimicking the function of factor VIII is initiated. This medication most likely works by which of the following mechanisms?
CorrectIncorrect -
Question 4 of 38
4. Question
A 44-year-old male is hospitalized with unstable angina. On the fourth day of hospitalization he develops severe foot pain and right toe paleness. His laboratory testing is significant for platelet count of 60,000/mm3. Treatment with argatroban is initiated immediately. The drug (argatroban) used to treat this patient’s current condition has which mechanism of action?
CorrectIncorrect -
Question 5 of 38
5. Question
Cancer cells from a 23-year-old male with testicular cancer demonstrate high sensitivity to etoposide. The etoposide-killed cancer cells show a high number of double-stranded DNA fractures. These fractures are due to dysfunction of which of the following?
CorrectIncorrect -
Question 6 of 38
6. Question
A 52-year-old man comes to the emergency department (ED) due to 2 hours of burning substernal pain. Before coming to the ED, he took several tablets of antacid at home without any relief. The patient’s other medical problems include hypertension and hyperlipidemia. His temperature is 36.7 C (98 F), blood pressure is 160/90 mm Hg, and pulse is 92/min. Cardiovascular examination shows normal S1 and S2 without any murmurs. There is no abdominal tenderness. ECG shows ST-segment depression in leads II, III, and aVF. Troponin is 0.06 ng/mL (normal <0.01 ng/mL). As part of this patient’s treatment, enoxaparin therapy is initiated. This drug is expected to bind to which of the following substances in this patient’s blood?
CorrectIncorrect -
Question 7 of 38
7. Question
A 64-year-old woman is hospitalized for elective hip replacement surgery. She has osteoarthritis and is experiencing progressive right hip pain that limits her daily activities. The patient also has a history of hypertension and hypothyroidism. Her blood pressure is 130/80 mm Hg and pulse is 90/min. BMI is 31.5 kg/m2. Her serum creatinine is 0.9 mg/dL. She undergoes surgery without complications and is postoperatively started on a medication that is a low molecular weight fraction of a negatively charged chemical stored in mast cell granules. Which of the following is the most likely mechanism of action of the medication used?
CorrectIncorrect -
Question 8 of 38
8. Question
A 57-year-old man comes to the emergency department due to fevers and painful mouth ulcers. He recently began methotrexate therapy for psoriasis. However, the patient has mistakenly taken the medication daily instead of 3 times a week. His past medical history is also significant for hypertension and chronic kidney disease. Temperature is 38.1 C (100.6 F). Examination reveals multiple aphthous ulcers in the oral pharynx. A complete blood cell count is as follows:
Hemoglobin
10.4 g/dL
Platelets
120,000/mm3
Leukocytes
2,800/mm3
Which of the following is the best next step in management?
CorrectIncorrect -
Question 9 of 38
9. Question
A 56-year-old woman comes to the emergency department due to 3 days of frequent urination, suprapubic pain, dysuria, and progressive hematuria. She has had no fevers or chills. The patient has a history of lymph-node-positive breast cancer that was diagnosed following a routine mammogram. A month ago, she began treatment with systemic chemotherapy. Temperature is 37.1 C (98.8 F). Suprapubic tenderness is present on abdominal examination. Hemoglobin is 9.8 g/dL. Urinalysis shows numerous red blood cells but no leukocyte esterase or bacteria. Which of the following could have prevented this patient’s current condition?
CorrectIncorrect -
Question 10 of 38
10. Question
A 56-year-old man comes to the office due to a neck mass, which has been enlarging over the past 2 months. The patient also reports night sweats and a weight loss of 7 kg (15.4 lb). Physical examination shows a 3-cm, rubbery, nontender cervical lymph node. Biopsy of the neck mass reveals non-Hodgkin lymphoma. The patient undergoes systemic chemotherapy with vincristine as part of the regimen. Which of the following adverse effects is most likely to occur with use of this medication?
CorrectIncorrect -
Question 11 of 38
11. Question
A 70-year-old man develops progressive fatigue and easy bruising. He has had unintentional weight loss and episodic fevers. Physical examination shows splenomegaly and enlarged lymph nodes in the cervical, axillary, and inguinal regions. Complete blood count reveals anemia, thrombocytopenia, and leukocytosis. Peripheral blood smear shows an increased number of small, mature-appearing, abnormal lymphocytes, which express CD20 and kappa light chains. During treatment, the patient receives an agent that directly binds and inhibits BCL-2 protein. Which of the following is the most likely effect of this therapy on the abnormal cells?
CorrectIncorrect -
Question 12 of 38
12. Question
A 23-year-old woman with a history of sickle cell disease comes to the office for follow-up. The patient is homozygous for hemoglobin S and has had frequent episodes of painful vasoocclusive crises. She has no other medical conditions and takes folic acid supplementation. The patient does not use tobacco, alcohol, or illicit drugs. Physical examination shows no abnormalities. Additional therapy with hydroxyurea is planned. This medication is likely to improve this patient’s symptoms primarily through which of the following mechanisms?
CorrectIncorrect -
Question 13 of 38
13. Question
A 57-year-old female presents to the emergency department after vomiting a substance that resembled “coffee grounds.” She reports that she now feels lightheaded as well. Her past medical history is significant for deep venous thromboses, for which she takes warfarin, and occasional joint pain, for which she takes aspirin. Her blood pressure is 90/60 mm Hg and her pulse is 110 beats per minute. Which of the following substances would provide the fastest reversal of warfarin’s effects?
CorrectIncorrect -
Question 14 of 38
14. Question
A 72-year-old woman is brought to the emergency department due to severe shortness of breath, tachypnea, and pleuritic-type chest pain. She had a similar episode 3 weeks ago and has been treated with warfarin since then. The patient has several other medical conditions and is taking all her medications as recommended. On examination, her temperature is 36.7 C (98 F), blood pressure is 120/60 mm Hg, pulse is 118/min, and respirations are 28/min. Pulse oximetry is 93% on room air. Laboratory results are notable for the following:
Coagulation studies
Prothrombin time (PT)
18 sec
International Normalized Ratio (INR)
1.6
The patient’s INR is subtherapeutic compared to her target INR range of 2.0-3.0 with warfarin. This patient is most likely taking which of the following additional medications?
CorrectIncorrect -
Question 15 of 38
15. Question
A 63-year-old man comes to the hospital due to sudden onset chest pain and dyspnea. He has a history of glioblastoma multiforme, for which he has been undergoing radiation treatment. His temperature is 36.1 C (97 F), blood pressure is 110/80 mm Hg, pulse is 118/min, and respirations are 26/min. Pulse oximetry shows 90% on room air. A CT pulmonary angiogram demonstrates a left-sided pulmonary embolism. The patient is started on continuous unfractionated heparin infusion. Several hours later, the patient begins to experience severe headache and quickly becomes unconscious. A CT of the head reveals bleeding into the tumor. After stopping the heparin infusion, administration of which of the following is indicated for immediate anticoagulant reversal?
CorrectIncorrect -
Question 16 of 38
16. Question
A 12-year-old boy has prolonged oral bleeding immediately after a tooth extraction. Despite several interventions, the bleeding persists for hours and stops only after desmopressin (DDAVP) administration. The patient has several dental caries due to excessive juice intake and inconsistent oral hygiene. He has no other medical problems and takes no medications. His father and paternal grandmother have also had excessive bleeding after dental procedures. Review of systems is positive for mild bruising on his legs. Which of the following is the most likely therapeutic mechanism of desmopressin for this patient’s condition?
CorrectIncorrect -
Question 17 of 38
17. Question
A 62-year-old woman comes to the office due to a month of progressive right chest and back pain. The patient also noticed a right breast lump several months ago, which has grown over time. She has no prior medical issues, has not seen a physician for many years, and has had no cancer screenings. Physical examination reveals a hard, 5-cm right breast mass, enlarged axillary lymph nodes, and point tenderness along the right-sided ribs and 10th thoracic vertebra. Biopsy of the breast mass shows hormone receptor–positive, HER2-negative, invasive ductal carcinoma. Skeletal survey demonstrates lytic lesions of the ribs and thoracic vertebrae. As part of the treatment regimen, the patient receives a medication that is a potent inhibitor of cyclin-dependent kinase. Which of the following is the most likely dose-limiting toxicity of this medication?
CorrectIncorrect -
Question 18 of 38
18. Question
A 61-year-old man comes to the hospital due to 2 days of right leg swelling. He recently flew back to the United States from Japan, where he was visiting his daughter who is spending a semester abroad. The patient has a medical history of hypertension and hyperlipidemia. Temperature is 37.2 C (99 F), blood pressure is 136/70 mm Hg, and pulse is 90/min. Pulse oximetry shows 99% on room air. Cardiopulmonary examination is within normal limits. There is tenderness and swelling of the right lower extremity up to the mid thigh. Laboratory studies, including prothrombin time and activated partial thromboplastin time, are normal. Venous doppler reveals an occluding thrombus in the right femoral and popliteal veins. The patient is started on rivaroxaban. This drug inhibits which of the following?
CorrectIncorrect -
Question 19 of 38
19. Question
Two antineoplastic drugs are shown to inhibit intracellular thymidylate formation. The chemotherapeutic effect of drug X can be overcome by N5-formyl-tetrahydrofolate supplementation, but that of drug Y is not affected. The drugs described in this scenario are most likely which of the following?
CorrectIncorrect -
Question 20 of 38
20. Question
A 5-year-old boy is brought to the emergency department due to an hour of epistaxis after he was elbowed in the face during a basketball game. He has a history of prolonged bleeding from the gums after brushing his teeth. Vital signs are normal. The patient has scattered bruises along his arms and legs. He receives desmopressin, which stops the bleeding. This treatment most likely improved the patient’s condition through which of the following therapeutic mechanisms?
CorrectIncorrect -
Question 21 of 38
21. Question
An elderly patient is diagnosed with aggressive non-Hodgkin’s lymphoma. The tumor cells are shown to stain strongly for the CD20 marker. Which of the following biologic agents can be added to this patient’s chemotherapy regimen to help improve the treatment response?
CorrectIncorrect -
Question 22 of 38
22. Question
A 63-year-old man comes to the emergency department because of 1 hour of sudden-onset chest pain. He developed the pain after eating dinner and did not have relief with antacids. His other medical problems include hypertension, hyperlipidemia, and type 2 diabetes mellitus. In the emergency department, his blood pressure is 100/60 mm Hg, pulse is 100/min, and respirations are 18/min. His EKG shows ST-segment depression in the inferior leads, and his cardiac serum troponin T is positive. The following anticoagulants are being considered for treatment:
Anticoagulant
Molecular Weight
Unfractionated heparin
5000-30,000 u
Enoxaparin
3000-8000 u
Fondaparinux
Pentasaccharide
Which of the following anticoagulants is most effective in inactivating thrombin?
CorrectIncorrect -
Question 23 of 38
23. Question
A 24-year-old woman, gravida 1 para 0, at 14 weeks gestation, comes to the emergency department due to left leg swelling and pain for the past 2 days. She has had no chest pain or shortness of breath. The patient has no prior medical conditions and her only medication is a prenatal vitamin. She is a lifetime nonsmoker. Physical examination shows 1+ edema of the left lower extremity to the knee, associated with mild erythema. The left calf diameter measures 3 cm greater than the right. Doppler ultrasonography shows left popliteal and femoral vein thrombosis. Serum creatinine level is 0.7 mg/dL. Which of the following is the most appropriate pharmacotherapy for this patient?
CorrectIncorrect -
Question 24 of 38
24. Question
A 12-year-old boy is evaluated due to fever, fatigue, and easy bruising. Physical examination shows pallor, generalized lymphadenopathy, and bone tenderness. Laboratory evaluation reveals anemia, thrombocytopenia, and leukocytosis with atypical lymphoblasts. Bone marrow biopsy is consistent with acute lymphoblastic leukemia. The patient is treated with combination chemotherapy, and 6-mercaptopurine is administered for maintenance of remission. Which of the following enzymes inactivates this drug?
CorrectIncorrect -
Question 25 of 38
25. Question
A 72-year-old woman comes to the office for a routine follow-up appointment. She has no symptoms and her past medical history is insignificant. Her temperature is 36.7 C (98 F), blood pressure is 110/80 mm Hg, and pulse is 76/min and irregular. ECG shows atrial fibrillation with no ischemic changes. Anticoagulation therapy with warfarin is initiated for stroke prevention. Two days later, the patient is hospitalized with severe skin and subcutaneous fat necrosis. Drug effects on which of the following processes are most likely responsible for this patient’s skin findings?
CorrectIncorrect -
Question 26 of 38
26. Question
A 26-year-old woman, gravida 2 para 1, at 8 weeks gestation comes to the office due to pain and swelling of her left leg for the past day. The patient had a pulmonary embolism during her previous pregnancy, and prophylactic low-molecular-weight heparin therapy was initiated 6 days ago. She has no other medical conditions and takes prenatal vitamins. Physical examination shows left lower extremity edema and calf tenderness but no other abnormalities. Venous duplex ultrasonography reveals acute left femoral vein thrombosis. Platelet count, which was normal prior to anticoagulant therapy initiation, is 84,000/mm3. Other blood cell counts and renal and liver function studies are within normal limits. Which of the following most likely predisposed this patient to her current condition?
CorrectIncorrect -
Question 27 of 38
27. Question
A 61-year-old man comes to the office due to 4 months of easy fatigability, anorexia, and a 4.5-kg (10-lb) unintentional weight loss. His stool occult blood testing is positive, and laboratory studies show microcytic hypochromic anemia. Colonoscopy reveals a 6-cm (2.4-in) mass in the descending colon. Biopsy samples obtained during the procedure are consistent with adenocarcinoma. A subsequent CT scan of the abdomen shows multiple metastatic liver lesions. Therapy with monoclonal antibodies that bind to epidermal growth factor receptors on the malignant cells is considered. An activating mutation in which of the following will most likely make this therapy ineffective?
CorrectIncorrect -
Question 28 of 38
28. Question
A 50-year-old woman comes to the emergency department due to a large, painful skin lesion. She does not remember sustaining any trauma. She was recently diagnosed with atrial fibrillation and started on treatment with warfarin. The patient noticed the lesion about a day and a half after starting the medication. Temperature is 36.7 C (98 F), blood pressure is 130/82 mm Hg, and pulse is 88/min and irregularly irregular. Physical examination findings are shown in the image below.
The remainder of the skin examination is normal. Which of the following is the most likely cause of this patient’s skin lesion?
CorrectIncorrect -
Question 29 of 38
29. Question
A 76-year-old man is brought to the emergency department with severe midsternal chest pain and diaphoresis. Past medical history is significant for hypertension, type 2 diabetes mellitus, and asymptomatic right carotid artery stenosis. His blood pressure is 120/70 mm Hg and pulse is 75/min. Lungs are clear on auscultation. ECG shows ST segment elevations greater than 1 mm in leads II, III, and aVF. The patient receives aspirin immediately upon arrival followed by alteplase and a low-dose beta blocker. A single dose of intravenous morphine is given for pain control. Several hours later, the patient is found to be comatose with asymmetric pupils and an irregular breathing pattern. What is the most likely cause of this patient’s current condition?
CorrectIncorrect -
Question 30 of 38
30. Question
A 68-year-old man comes to the office due to thigh and leg pain that worsens with exertion. He is unable to walk through the local mall with his wife without discomfort. Past medical history is significant for hypertension and diabetes mellitus. The patient smokes 2 packs of cigarettes a day and consumes alcohol occasionally. Physical examination of the extremities shows weak dorsalis pedis pulses in both feet. Further evaluation confirms moderate peripheral arterial disease involving both lower extremities. Which of the following drugs would best provide symptomatic improvement due to direct dilation of arteries and inhibition of platelet aggregation?
CorrectIncorrect -
Question 31 of 38
31. Question
A 64-year-old man comes to the office due to exertional chest pain over the last 6 months. He is a lifelong 1 pack per day cigarette smoker and has a history of type 2 diabetes mellitus and peripheral artery disease. The patient undergoes treadmill exercise stress testing and develops substernal chest pain on moderate exertion accompanied by ECG changes that resolve immediately upon rest. He refuses invasive cardiac testing. The patient is started on low-dose aspirin therapy for secondary prevention of cardiovascular disease but experiences shortness of breath and wheezing with the medication. Which of the following is the best alternate therapy for this patient?
CorrectIncorrect -
Question 32 of 38
32. Question
A 24-year-old woman comes to the emergency department due to bloody emesis. She had 2 episodes of vomiting bright red blood and feels lightheaded and dizzy. The patient was recently diagnosed with factitious disorder after being hospitalized multiple times for a myriad of symptoms and undergoing several invasive procedures. Physical examination shows scattered ecchymoses. The abdomen is soft and nontender. Rectal examination shows maroon-colored, guaiac-positive stool. After much questioning, she admits to having ingested rat poison several days ago. Immediate treatment of this patient should include which of the following?
CorrectIncorrect -
Question 33 of 38
33. Question
A 68-year-old man comes to the emergency department due to lightheadedness, generalized weakness, and palpitations. The patient has a prolonged history of hypertension and takes amlodipine. He does not use tobacco or alcohol. His blood pressure is 110/60 mm Hg and pulse is 144/min and irregular. ECG shows an irregularly irregular rhythm and absent P waves. The patient is started on beta blocker therapy for rate control, with improvement in his heart rate. Long-term anticoagulation is initiated to prevent atrial thrombus formation, and he is eventually discharged home on warfarin. Which of the following is the best test to monitor the anticoagulation effect of warfarin in this patient?
CorrectIncorrect -
Question 34 of 38
34. Question
A 45-year-old man is brought to a rural emergency department due to severe chest pain, sweating, and nausea. The symptoms began suddenly an hour ago. He has no significant medical history. His father died at age 50 after experiencing sudden-onset chest pain. The patient smokes a pack of cigarettes daily. He does not take any medications and has no known drug allergies. Examination shows normal heart sounds and breath sounds. ECG shows sinus tachycardia with ST segment elevation in leads II, III, and aVF. Medical management for the patient’s acute condition is initiated. After initial treatment, the chest pain decreases in intensity and a reperfusion complex ventricular arrhythmia emerges. The arrhythmia is asymptomatic and resolves spontaneously. Which of the following drugs is most likely responsible for rapid reperfusion in this patient?
CorrectIncorrect -
Question 35 of 38
35. Question
A 34-year-old man diagnosed with advanced non-Hodgkin lymphoma undergoes doxorubicin-containing chemotherapy with good clinical response. Several weeks after the last chemotherapy cycle, the patient comes to the office with progressive exertional dyspnea. He has difficulty sleeping flat at night and needs to use multiple pillows to fall asleep. The patient is a lifetime nonsmoker and has no family history of heart disease. Blood pressure is 124/72 mm Hg, pulse is 86/min, and respirations are 16/min. Oxygen saturation on room air is 96%. Which of the following is most likely responsible for this patient’s current symptoms?
CorrectIncorrect -
Question 36 of 38
36. Question
An investigator is studying a line of tumor cells that has developed resistance to several anticancer agents. The tumor cells are exposed to various chemotherapeutic agents, and the intracellular drug concentration is subsequently measured. Following exposure to doxorubicin, the intracellular drug concentration is reduced compared to control cells with no doxorubicin resistance. Western blot analysis shows the tumor cells have increased expression of a specific glycoprotein. This protein most likely has which of the following functions?
CorrectIncorrect -
Question 37 of 38
37. Question
A 52-year-old man comes to the office for an annual preventive visit. He has a history of seasonal allergies, and has recently started taking a medication that has improved his runny nose, sneezing, and watery eyes. Medical history is remarkable for diet-controlled type 2 diabetes mellitus and eczema. He is not on any other medications. He does not use tobacco, alcohol, or illicit drugs. Examination shows a well-appearing man with flushed cheeks and dilated pupils. This patient’s physical examination findings are best explained by which of the following mechanisms?
CorrectIncorrect -
Question 38 of 38
38. Question
A 67-year-old man is evaluated for persistent lower back pain and fatigue. The patient has no other medical conditions and takes no medications. Laboratory testing shows anemia and renal dysfunction. Serum total protein levels are elevated, and electrophoresis of the serum proteins reveals a narrow spike in the gamma globulin region. A subsequent bone marrow biopsy shows abnormal marrow cells. A medication that blocks cellular proteasome action is administered. This treatment is most likely to cause which of the following effects on the abnormal cells?
CorrectIncorrect